Kap 800 Question

This topic has expert replies
Master | Next Rank: 500 Posts
Posts: 184
Joined: Thu Nov 25, 2010 9:57 pm
Thanked: 1 times
Followed by:5 members
GMAT Score:700

Kap 800 Question

by chaitanya.mehrotra » Sat Jul 30, 2011 8:31 am
A wave of incidents of unusual violence, from murder to acts of self-destruction, plagued the small medieval town for a period of five years, nearly wiping out the population. At the same time, there was an unusual shift in the area's weather pattern. Rainfall was so heavy and continuous that the wheat crop probably fell prey to the ergot fungus. When eaten, grain thus affected can cause ergotism, a disease associated with hallucinations and other disturbing psychological side effects. In the end we can conclude that the violence was the result of freakish weather conditions.

Which of the following is the most effective rebuttal to the contention made above?
(A) It is based upon a series of plausible suppositions rather than upon contemporary evidence.
(B) No clear distinction is drawn between cause and effect.
(C) Explanations of historical events cannot be convincing when too great a role is assigned to chance or the irrational.
(D) The author makes no distinction between probable occurrence and actual occurrence.
(E) Such crucial terms as "unusual violence" are not adequately defined in regard to the specific historical event.

[spoiler]OA D , doubt why is A wrong . How does the word "contemporary evidence" needs to be interpreted here ? [/spoiler]

User avatar
Legendary Member
Posts: 965
Joined: Thu Jan 28, 2010 12:52 am
Thanked: 156 times
Followed by:34 members
GMAT Score:720

by vineeshp » Sat Jul 30, 2011 10:50 am
Contemporary evidence: evidence available today. A is wrong because author has used evidence and not suppositions.

I did not arrive at D, but POE left me with only D.
How does the word "contemporary evidence" needs to be interpreted here ?
You must use need not needs. Also, the right and best way to put it is:
How do I interpret "contemporary evidence"? :)
Vineesh,
Just telling you what I know and think. I am not the expert. :)

User avatar
Master | Next Rank: 500 Posts
Posts: 407
Joined: Tue Jan 25, 2011 9:19 am
Thanked: 25 times
Followed by:7 members

by Ozlemg » Sat Jul 30, 2011 12:37 pm
chaitanya.mehrotra wrote:A wave of incidents of unusual violence, from murder to acts of self-destruction, plagued the small medieval town for a period of five years, nearly wiping out the population. At the same time, there was an unusual shift in the area's weather pattern. Rainfall was so heavy and continuous that the wheat crop probably fell prey to the ergot fungus. When eaten, grain thus affected can cause ergotism, a disease associated with hallucinations and other disturbing psychological side effects. In the end we can conclude that the violence was the result of freakish weather conditions.

Which of the following is the most effective rebuttal to the contention made above?
(A) It is based upon a series of plausible suppositions rather than upon contemporary evidence.
(B) No clear distinction is drawn between cause and effect.
(C) Explanations of historical events cannot be convincing when too great a role is assigned to chance or the irrational.
(D) The author makes no distinction between probable occurrence and actual occurrence.
(E) Such crucial terms as "unusual violence" are not adequately defined in regard to the specific historical event.

[spoiler]OA D , doubt why is A wrong . How does the word "contemporary evidence" needs to be interpreted here ? [/spoiler]
why not B? Doesnt B rephrase as "there may not be cause and effect btw 2 events"
The more you suffer before the test, the less you will do so in the test! :)

User avatar
GMAT Instructor
Posts: 2193
Joined: Mon Feb 22, 2010 6:30 pm
Location: Vermont and Boston, MA
Thanked: 1186 times
Followed by:512 members
GMAT Score:770

by David@VeritasPrep » Sun Jul 31, 2011 8:58 am
I received a PM for this one:

It is an unusual question, and not likely to be on the GMAT in this exact form, but some of the questions in our own Veritas books are designed to stretch your thinking as well. So let's give this one a shot!

First of all, I would classify this as a flawed reasoning question. The actual question stem here says "Which of the following is the most effective rebuttal to the contention made above?" However the answer choices are not actually rebuttals but are descriptions of flaws. On both the GMAT and even the LSAT the question stem really should read, "Which of the following choices best describes the flaw in the reasoning above."

So we will approach this as a FLAWED REASONING question. We want a description of the logical problem that this argument has...

When I read a problem like this I am really looking for shifts in language, unexpected things said, for example perhaps the argument tells us that "Tennis is one of the sports where female athletes are sometimes more popular than male athletes." Perhaps the conclusion would then say "Therefore tennis is the only sport where companies might be better off sponsoring female athletes."

Did you spot the two shifts in language??

First, we went from "one of the sports" in the evident to "the only sport" in the conclusion...so that is a problem.

Second, we went from "female athletes are sometimes more popular" to "companies might be better off sponsoring..." These are not the same things. Maybe there are reasons why companies would choose to sponsor a female athlete - or a male athlete - that have nothing to do with being the most popular.

Watch for these sorts of shifts. Basically I say watch for the argument to say something a little Unexpected that did not have to be said.

What about this argument? Well we have these two premises:
Rainfall was so heavy and continuous that the wheat crop probably fell prey to the ergot fungus.
and
When eaten, grain thus affected can cause ergotism,
Do you see that each is just a possibility or probability?

That is why I found the conclusion to be a little unexpected. It says:
In the end we can conclude that the violence was the result of freakish weather conditions.
Not "possibly" nor "probably" but "definitely" "the result of freakish weather conditions."

So that is the flaw right there.
It is like playing the lottery or other gambling. You have a chance to win, but that is not the same thing as you will win.

That is choice D. The author fails to distinguish between the "probable" in the evidence (premises) and the "actual" in the conclusion.


As to the use of "contemporary evidence" is would say that this probably means "contemporary to the event." So I would say that in that context answer choice A means, "It is based on some likely assumptions that we are making now, rather than good evidence from that time period (such as ancient writings, etc.)

"Contemporary" is most often used to mean "now" such as contemporary music or contemporary furniture but in this case I think it means "contemporaneous" or "contemporaneously" meaning "Originating, existing, or happening during the same period of time" So that would mean evidence from the same period of time as the medieval violence.

Like most questions, including the ones that I write, a little editing would not hurt, but D works as the correct answer here.
Veritas Prep | GMAT Instructor

Veritas Prep Reviews
Save $100 off any live Veritas Prep GMAT Course

Senior | Next Rank: 100 Posts
Posts: 67
Joined: Thu Jun 26, 2008 8:34 am
Location: india
Thanked: 1 times

by dinaroneo » Sun Jul 31, 2011 4:28 pm
Thanks! Its clear: D should be the answer.